Đến nội dung

trieutuyennham nội dung

Có 461 mục bởi trieutuyennham (Tìm giới hạn từ 08-05-2020)



Sắp theo                Sắp xếp  

#690511 Xin tổng hợp về tổ hợp liên quan đến trò chơi

Đã gửi bởi trieutuyennham on 14-08-2017 - 19:08 trong Tổ hợp và rời rạc

Chào mọi người, mình post bài viết này mong mọi người ủng hộ để có thể sưu tập được các bài toán tổ hợp liên quan đến trò chơi, luật chơi hay thuật toán để dành chiến thắng.

Mong ban quản trị không khóa bài viết ạ.

Cảm ơn mọi người.

Ai có bài toán nào vui lòng trả lời ở dưới bài viết hộ mình luôn nhé!

Bài toán:

Trên mặt đất có n viên sỏi, 2 người chơi nhặt lần lượt không quá k viên $(n,k\in N;1\leq k\leq n)$. Người thua cuộc là người đến lượt mình khoog còn viên sỏi nào để nhặt.Hỏi ai sẽ thắng và cách chơi như thế nào? 

Cũng hỏi như trên khi thay k bởi 1 hoặc 1 số nguyên tố

                             hoặc thay k bởi $2^{k}$




#696699 x=$\sqrt[7]{\frac{3}{5}}+\s...

Đã gửi bởi trieutuyennham on 16-11-2017 - 18:10 trong Phương trình - hệ phương trình - bất phương trình

Tìm đa thức bậc 7 có hệ số nguyên nhận x=$\sqrt[7]{\frac{3}{5}}+\sqrt[7]{\frac{5}{3}}$ làm 1 nghiệm

Đặt $a=\sqrt[7]{\frac{3}{5}};b=\sqrt[7]{\frac{3}{5}}\Rightarrow ab=1;a+b=x$

ta có

$a^{3}+b^{3}=x^{3}-3x;a^{4}+b^{4}=x^{4}-4x^{2}+2$

$(a^{4}+b^{4})(a^{3}+b^{3})=a^{7}+b^{7}+(ab)^{3}(a+b)$

$\Leftrightarrow (x^{3}-3x)(x^{4}-4x^{2}+2)=\frac{3}{5}+\frac{5}{3}+x\Leftrightarrow 15X^{7}-105x^{5}+210x^{3}-105x-34=0$

Từ đây suy ra đa thức cần tìm




#694333 Trong một tam giác đều cạnh 1, ta đặt 17 điểm. Chứng minh rằng, tồn tại hai đ...

Đã gửi bởi trieutuyennham on 07-10-2017 - 21:10 trong Số học

Trong một tam giác đều cạnh 1, ta đặt 17 điểm. Chứng minh rằng, tồn tại hai điểm mà khoảng cách giữa chúng nhỏ hơn $\frac{1}{4}$.

Chia tam giác đó thành 16 tam giác đều bằng nhau cạnh $\frac{1}{4}$. Theo Dirichlet tồn tại 2 điểm cùng thuộc 1 tam giác và khoảng cách giữa chúng không lớn hơn $\frac{1}{4}$




#691239 toán về diện tích lớp 9

Đã gửi bởi trieutuyennham on 21-08-2017 - 21:02 trong Hình học

Xét ngũ giác ABCDE có F là giao của AD và CE

Dễ dàng cm tứ giác ACBF là hình bình hành

Ta có

$S_{ABC}=S_{BCD}=S_{CDE}=S_{DEA}=S_{AEB}=S_{AFC}=1$

Đặt $S_{AEF}=x(x>0)$

$\frac{S_{AEF}}{S_{EFD}}=\frac{x}{1-x}=\frac{AF}{FD}=\frac{S_{AFC}}{S_{CFD}}=\frac{1}{x}$

Dễ dàng tính được $x=\frac{\sqrt{5}-1}{2}$

$\Rightarrow S_{ABCDE}=\frac{5+\sqrt{5}}{2}$ 




#691549 Toán vận tốc

Đã gửi bởi trieutuyennham on 26-08-2017 - 06:36 trong Số học

một oto đi từ A đến B vs vận tốc 30km/h;Đi từ B về A vs vận tốc 20km/h.Hỏi vận tốc trung bình cả quãng đường là bao nhiêu?

Vận tốc trung bình 

$\frac{2}{\frac{1}{20}+\frac{1}{30}}=24$




#688118 Toán thi Học sinh giỏi và Olympic

Đã gửi bởi trieutuyennham on 20-07-2017 - 09:42 trong Hướng dẫn - Trợ giúp - Giải đáp thắc mắc khi sử dụng Diễn đàn

Tổng của các số chia hết cho 3 từ 20000 đến 2000000 là:66659970000

Tổng của các số chia hết cho 15 từ 20000 đến 2000000 là 133331533005

$\Rightarrow$ Tổng của các số chia hết cho 3 nhưng không chia hết cho 5 là 533268136995 




#690038 Toán số học lớp 9

Đã gửi bởi trieutuyennham on 09-08-2017 - 21:00 trong Số học

Chứng minh rằng tồn tại một số là bội của 19 có tổng các chữ số bằng 19.

Tồn tại một số k sao cho 

$10^{k}-1;10^{2k}-1;...;10^{19k}-1\vdots 19$

$\Rightarrow 10^{k}+10^{2k}+...+10^{19k}-19\vdots 19$

$\Rightarrow 10^{k}+10^{2k}+...+10^{19k}\vdots 19$

suy ra đpcm




#690047 Toán số học lớp 9

Đã gửi bởi trieutuyennham on 09-08-2017 - 21:26 trong Số học

10^{k} -1 không chia hết cho 19 mà bạn 

Bạn có thể áp dụng nguyên lý diricle vào bài này ko ?

mình chỉ nói là tồn tại không phải là mọi số chẳng hạn $k=18$ 

hình như bài này vẫn có thể dùng Dirichlet




#690067 Toán số học lớp 9

Đã gửi bởi trieutuyennham on 09-08-2017 - 22:21 trong Số học

Cho 7 số tự nhiên bất kì. CMR luôn chọn được 4 số có tổng chia hết cho 4

bạn có thể tham khảo tại đây

http://giasuducminh....Nhien-a149.html




#687542 Toán khai phương một tích phần 2

Đã gửi bởi trieutuyennham on 14-07-2017 - 18:42 trong Đại số

3)

Ta có $\frac{a^{2}}{b-1}+4(b-1)+\frac{b^{2}}{a-1}+4(a-1)\geq 4a+4b$

$\Rightarrow K\geq 8$




#687650 Toán chia hết phương pháp xét số dư

Đã gửi bởi trieutuyennham on 15-07-2017 - 21:35 trong Đại số

Giả sử a;b là 2 số trong 100 số 

a=6p+r;b=6q+m($m;r;p;q\in N;m;r\leq 5$)

Vì a+b chia hết cho 6 nên m+r chia hết cho 6 nên $m+r\in \left \{ \right.0;6\left. \right \}$

$\Rightarrow$ (m;r) có dạng $(0;0);(1;5);(2;4);(3;3)$

Gọi c là số thứ 3  trong những số chọn ra $(c\neq a;b)$ và c=6k+t $(k;n\in N;n\leq 5)$

Vì a+c chia hết cho 6 nên lập luận tương tự (r;t) có dạng như trên

$\Rightarrow m=t$ Như vậy nếu (r;t) và (r;m) có dạng (1;5);(2;4) và các hoán vị  thì m+t không chia hết cho 6

Nên (r;t);(r;m) chỉ có thể là (0;0);(3;3)

$\Rightarrow$ Tất cả n số chọn ra đều chia hết cho 6 hoặc chia 6 dư 3

Từ 1 đến 100 có 16 số chia hết cho 6

và 17 số chia 6 dư 3 nên n lớn nhất là 17 




#691661 Toán 9 đường tròn

Đã gửi bởi trieutuyennham on 27-08-2017 - 10:37 trong Hình học

Cho tam giác ABC có 3 góc nhọn, các đường cao BD và CE cắt nhau tại F. Kẻ BM,CH lần lượt vuông góc với ED và DE. Cho B,C cố định, A thay đổi. Tìm điều kiện của tam giác ABC để HM lớn nhất.

Kẻ BK vuông góc với CH 

Tứ giác BMHK là hình chữ nhật nên

$HM=BK\leq BC$




#687468 Toán 7 Trị Tuyệt đối

Đã gửi bởi trieutuyennham on 14-07-2017 - 07:34 trong Đại số

Ta có 

$B=\left | x-1 \right |+\left | x-2 \right |+...+\left | x-100 \right |$=$(\left | x-1 \right |+\left | 100-x \right |)+...+(\left | x-50 \right |+\left | 51-x) \right |$$\geq 101.50=5050$

Đẳng thức xảy ra $\Leftrightarrow 50\leq x\leq 51$




#687452 Toán 7 Trị Tuyệt đối

Đã gửi bởi trieutuyennham on 13-07-2017 - 21:41 trong Đại số

1b)

Do x<y<1 nên $\left | x-1 \right |-\left | y-1 \right |=(1-x)-(1-y)=-x+y$

nên x-y=-50




#687444 Toán 7 Trị Tuyệt đối

Đã gửi bởi trieutuyennham on 13-07-2017 - 19:39 trong Đại số

2)

Do $\left | xy+\frac{2}{3} \right |\geq 0;\left | yz-\frac{8}{9} \right |\geq 0;\left | zx+\frac{3}{4} \right |\geq 0$ nên $\left\{\begin{matrix} & xy=-\frac{2}{3} & \\ & yz=\frac{8}{9} & \\ & zx=-\frac{3}{4} & \end{matrix}\right.$

Nhân vế theo vế ta được $xyz=\frac{2}{3}$ hoặc $xyz=-\frac{2}{3}$




#688511 Topic: Các bài toán về tính chia hết

Đã gửi bởi trieutuyennham on 24-07-2017 - 14:23 trong Số học

ta có

$n^{5}-11n=(n-2)(n-1)n(n+1)(n+2)-10n\vdots 5$




#693098 Topic: Các bài toán về tính chia hết

Đã gửi bởi trieutuyennham on 15-09-2017 - 20:49 trong Số học

Mọi ngưòi chỉ giúp:Tìm số tự nhiên n được viết bởi 1 số 1, 2 số 2,..., 9 số 9 và n là lập phương 1 số tự nhiên.

Tổng các chữ số của n là $1^{2}+2^{2}+...+9^{2}=285$ chia 9 dư 6 nên n chia 9 dư 6

Mà lập phương của 1 số tự nhiên không chia 9 dư 6 nên không tồn tại số n thỏa mãn




#696243 TOPIC thảo luận, trao đổi toán thi học sinh giỏi khối 10,11 .

Đã gửi bởi trieutuyennham on 08-11-2017 - 20:48 trong Chuyên đề toán THPT

Mình xin đóng góp 1 bài cho topic 

Cho các số thực $x,y,z$ thỏa mãn $0< x\leq y\leq z\leq 3$, $yz\leq 6,xyz\leq 6$. Tìm giá trị lớn nhất của $P=x+y+z$

ta có

$6=1+2+3=x(\frac{1}{x}+\frac{2}{y}+\frac{3}{z})+(y-x)(\frac{2}{y}+\frac{3}{z})+(z-y).\frac{3}{z}\geq x.3\sqrt[3]{\frac{6}{xyz}}+y.2\sqrt{\frac{6}{yz}}+(z-y).\frac{3}{z}\geq 3x+2(y-x)+z-y=x+y+z$

$\Rightarrow maxP=6\Leftrightarrow \left\{\begin{matrix} x=1\\ y=2\\ z=3 \end{matrix}\right.$




#696369 TOPIC thảo luận, trao đổi toán thi học sinh giỏi khối 10,11 .

Đã gửi bởi trieutuyennham on 11-11-2017 - 21:17 trong Chuyên đề toán THPT

Bài toán số 8(sưu tầm)

Cho A;B là các góc nhọn thỏa mãn $sinB=2005cos(A+B).sinA$

Tìm max tanB.

P/s mong mọi người ủng hộ topic




#696343 TOPIC thảo luận, trao đổi toán thi học sinh giỏi khối 10,11 .

Đã gửi bởi trieutuyennham on 11-11-2017 - 17:29 trong Chuyên đề toán THPT

Bài toán số 5 ( sưu tầm) : Tìm giới hạn : $\lim_{x\rightarrow 0}\frac{\sqrt{x^2-x+1}+\sqrt[4]{x^4-x+1}-2\sqrt[3]{x^3-x+1}}{x}$

 

Ta có

$\lim_{x\rightarrow 0}\frac{\sqrt{x^{2}-x+1}+\sqrt[4]{x^{4}-x+1}-2\sqrt[3]{x^{3}-x+1}}{x}=\lim_{x\rightarrow 0}\frac{(\sqrt{x^{2}-x+1}-1)+(\sqrt[4]{x^{4}-x+1}-1)-2(\sqrt[3]{x^{3}-x+1}-1)}{x}$

=$\lim_{x\rightarrow 0}(\frac{x-1}{\sqrt{x^{2}-x+1}+1}+\frac{x^{3}-1}{(\sqrt[4]{x^{4}-x+1}+1)(\sqrt[4]{(x^{4}-x+1)^{2}}+1)}-\frac{2(x^{2}-1)}{\sqrt[3]{(x^{3}-x+1})^{2}+\sqrt[3]{x^{3}-x+1}+1})=\frac{-1}{2}-\frac{1}{4}+\frac{2}{3}=\frac{-1}{12}$ (nhân liên hợp)

:D




#698260 TOPIC thảo luận, trao đổi toán thi học sinh giỏi khối 10,11 .

Đã gửi bởi trieutuyennham on 14-12-2017 - 21:09 trong Chuyên đề toán THPT

Bài toán số 31 (sưu tầm ) : Cho tam giác ABC , Tìm quỹ tích điểm M thỏa mãn $MA^2+2MB^2-3MC^2=k$ ( k là số thực tùy ý ) 

Gọi $I$ là điểm thỏa mãn $\overrightarrow{IA}+2\overrightarrow{IB}-3\overrightarrow{IC}=\overrightarrow{0}$

Ta có

$VT=(\overrightarrow{MI}+\overrightarrow{IA})^2+(\overrightarrow{MB}+2\overrightarrow{IB})^2-3(\overrightarrow{MC}+\overrightarrow{IC})^2$

=$IA^2+2IB^2-3IC^2=k$




#696562 TOPIC thảo luận, trao đổi toán thi học sinh giỏi khối 10,11 .

Đã gửi bởi trieutuyennham on 13-11-2017 - 21:08 trong Chuyên đề toán THPT

Bài toán số 8(sưu tầm)

Cho A;B là các góc nhọn thỏa mãn $sinB=2005cos(A+B).sinA$

Tìm max tanB.

P/s mong mọi người ủng hộ topic

1 cách giải khác như sau

Dùng công thức biến đổi tích thành tổng ta có

$sinB=2005cos(A+B).sinA=\frac{2005}{2}(sin(2A+B)+sin(-B))$

$\Rightarrow \frac{2007}{2}sinB=\frac{2005}{2}sin(2A+B)\Rightarrow sinB\leq \frac{2005}{2007}$

$\Rightarrow cosB\geq \sqrt{1-(\frac{2005}{2007})^{2}}$

Ta có

$tanB=\frac{sinB}{cosB}\leq \frac{\frac{2005}{2007}}{\sqrt{1-(\frac{2005}{2007})^{2}}}=\frac{2005\sqrt{2006}}{4012}$

Đẳng thức xảy ra $2A+B=90$  :D  :D




#690992 Topic giải phương trình vô tỉ.

Đã gửi bởi trieutuyennham on 19-08-2017 - 11:13 trong Phương trình, hệ phương trình và bất phương trình

3.$\sqrt{x^{2}-4x+8}+\sqrt[4]{2y^{2}+12y+19}=3$

 

$PT\Leftrightarrow \sqrt{(x-2)^2+4}+\sqrt[4]{2(y+3)^2+1}=3$

$\Rightarrow \left\{\begin{matrix} x=2 & \\ y=-3 & \end{matrix}\right.$




#698725 Topic bất đẳng thức

Đã gửi bởi trieutuyennham on 21-12-2017 - 21:40 trong Bất đẳng thức và cực trị

1.Chứng minh rằng với mọi n nguyên dương ta có:

   S=$\frac{1}{2}+\frac{1}{3\sqrt{2}}+\frac{1}{4\sqrt{3}}+...+\frac{1}{(n+1)\sqrt{n}}<\frac{5}{2}$

Ta có

$\frac{1}{(n+1)\sqrt{n}}=\frac{2}{2(n+1)\sqrt{n}}< \frac{2}{(n+1)\sqrt{n}+n\sqrt{n+1}}=2(\frac{1}{\sqrt{n}}-\frac{1}{\sqrt{n+1}})$

Áp dụng với n=1;2;,,,,ta được

$S< 2(1-\frac{1}{\sqrt{2}}+\frac{1}{\sqrt{2}}-\frac{1}{\sqrt{3}}+...+\frac{1}{\sqrt{n}}-\frac{1}{\sqrt{n+1}})< 2< \frac{5}{2}$




#698494 Topic BẤT ĐẲNG THỨC ôn thi vào lớp 10 THPT 2017 - 2018

Đã gửi bởi trieutuyennham on 17-12-2017 - 21:08 trong Bất đẳng thức và cực trị

Cho 1$\leq$a,b,c$\leq$2. Chứng minh: $(a+b+c)(\frac{1}{a}+\frac{1}{b}+\frac{1}{c})\leq 10$. < kỳ thi chuyên Trần phú 2013-2014>

tại đây

https://diendantoanh...rac1cleq-10036/